3x/2y=11 x+y/2=7 igualacion
Alguien me puede ayudar

Answers

Answer 1

La solución del sistema de ecuaciones es igual a (x, y) = (308/47, 42/47).

¿Cómo resolver un sistema de ecuaciones linear por igualación?

Los sistemas de ecuaciones basadas en entidades algebraicas se resuelven mediante procedimientos algebraicos. Grosso modo, el método de resolución por igualación consiste en despejar una variable en dos ecuaciones del sistema para eliminarla y reducir el número de ecuaciones lineales y el número de variables.

A continuación, presentamos una aplicación del método de eliminación:

[tex]\frac{3\cdot x}{2\cdot y} = 11[/tex]

[tex]x = \frac{22\cdot y}{3}[/tex]

[tex]x + \frac{y}{2} = 7[/tex]

[tex]x = 7 - \frac{y}{2}[/tex]

[tex]\frac{22\cdot y}{3} = 7 - \frac{y}{2}[/tex]

[tex]\frac{47\cdot y}{6} = 7[/tex]

y = 42/47

[tex]x = 7 - \frac{y}{2}[/tex]

x = 308/47

La solución del sistema de ecuaciones es igual a (x, y) = (308/47, 42/47).

Para aprender más sobre los sistemas de ecuaciones: https://brainly.com/question/15811265

#SPJ1


Related Questions

a box contains 7 red balls and 2 blue balls. how many more blue balls do you need to add to the box to make the probobility of pulling a red ball 0.5

A) 2
B) 3
C) 4
D) 5
E) None Of The Above

Answers

Using the probability concept, the number of blue balls you need to add is:

D) 5.

What is a probability?

A probability is given by the number of desired outcomes divided by the number of total outcomes.

A probability of 0.5 is reached if the number of red balls is the same as the number of blue balls. Current the difference is of 7 - 2 = 5, hence 5 blue balls must be added.

More can be learned about probabilities at https://brainly.com/question/14398287

#SPJ1

Who knows the answer for this problem please solve asap

Answers

The value of x to the nearest hundredth is 4.14

Side-angle-side theorem

The given diagram is a right triangle with the following parameters

Opposite side to 16 degrees = x

Hypotenuse = 15

Using the expression below

sin theta = x/15

sin16 = x/15

x = 15sin16

x = 4.135

Hence the value of x to the nearest hundredth is 4.14

Learn more on triangle here; https://brainly.com/question/2217700

#SPJ1

Consider the equation x+4=−2x+19. Let f(x)=x+4and g(x)=−2x+19. The graph of each function is shown. Coordinate plane with the graphs of two lines. The horizontal x axis labeled from negative three to nine in increments of one. The vertical y axis labeled from negative two to nineteen. The line f of x passes through ordered pairs zero comma four and two comma six. The line g of x passes through the ordered pairs zero comma nineteen and one comma seventeen. At what point do the graphs intersect? Enter your answer in the box.

Answers

The point of intersection of both graphs will have the coordinate (5, 9).

What is the Point of Intersection of the Graph?

We are given the functions;

f(x) = x + 4

g(x) = -2x + 19

Now, the point of intersection of both graphs is when both functions are equal which is at f(x) = g(x). Thus;

x + 4 = -2x + 19

x + 2x = 19 - 4

3x = 15

x = 15/3

x = 5

Thus;

f(x) = 5 + 4 = 9

g(x) = -2(5) + 19 = 9

Thus, the point of intersection of both graphs will have the coordinate (5, 9)

Read more about Graph Intersection at; https://brainly.com/question/11337174

#SPJ1

A function and its inverse are shown on the same graph. A function and its inverse are shown on the same graph.



Which statement describes the relationship between the function and its inverse?
The slope of f–1(x) is the same as the slope of f(x).
The slope of f–1(x) is the opposite as the slope of f(x).
The x-intercept of f–1(x) is the same as the y-intercept of f(x).
The x-intercept of f–1(x) is the opposite as the y-intercept of f(x).

Answers

Then the x-intercept of the inverse is the same as the y-intercept of f(x), this means that the correct statement is the third option.

Which statement is the correct one?

On the image, we can see two linear equations, such that the orange one is f(x) and the blue one is the inverse.

If you follow the orange line, you can see that the y-intercept of f(x) is y = 1.If you follow the blue line, you can see that the x-intercept of the inverse is x =1.

Then the x-intercept of the inverse is the same as the y-intercept of f(x), this means that the correct statement is the third option.

If you want to learn more about linear equations:

https://brainly.com/question/1884491

#SPJ1

Represent the following situation using Venn diagrams and answer the questions that follow. In a survey of 100 students in the institute language studies at oxford university, the number of students studying various languages in last four years were found to be : Italian 28, German 30,French 42,Italian and French 10, Italian and German 8, German and French 5, all the three languages 3. (1 point each) a. Venn diagram. b. How many students are studying none of the languages? c. How many students had French as their only language? d. How many students are studying at least one of the languages?​

Answers

There are 80 students studying at least one language

The Venn Diagram

The given parameters are:

Italian = 28German = 30French = 42Italian and French = 10Italian and German = 8German and French = 5Three languages = 3

Using the above parameters, we have:

German and French only = 5 - 3 = 2

Italian and French only = 10 - 3 = 7

Italian and German only = 8 - 3 = 5

French only = 42 - 7 - 2 - 3 = 30

German only = 30 - 2 - 5 - 3 = 20

Italian only = 28 - 7 - 5 - 3 = 13

See attachment for the Venn diagram

Students studying none of the languages

This is calculated as:

None + At least one = Total

Using the Venn diagram, we have:

None + (3 + 2 + 5 + 7 + 13 + 20 + 30) = 100

This gives

None + 80 = 100

Subtract 80 from both sides

None = 20

Hence, there are 20 students studying none of the languages

Students studying French only

This is calculated as

French only = French - French and Italian - French and German + All three

So, we have:

French only = 42 - 10 - 5 + 3

Evaluate

French only = 30

Hence, there are 30 students studying French only

Students studying at least one

In (b), we have:

None + At least one = Total ⇒ None + (3 + 2 + 5 + 7 + 13 + 20 + 30) = 100

By comparing both equations, we have

At least one = 3 + 2 + 5 + 7 + 13 + 20 + 30

This gives

At least one = 80

Hence, there are 80 students studying at least one language

Read more about Venn diagram at:

https://brainly.com/question/4910584

#SPJ1

3. The time for a swing to move forward and backward can be determined by the formula
T = 2√√√15₁
T represents the time, in seconds, taken by the swing to move through one
complete cycle (forward and back) and L represents the length of the rope supporting the swing.
9.8
Determine the length of the rope supporting a swing that takes 3.4 seconds to move through one
complete cycle. Show all steps and round the answer to the nearest tenth of a metre.

Answers

Step-by-step explanation:

T = 2×pi × sqrt(L/9.8)

now we know the time the swing should take for a complete back and forth cycle : 3 4 seconds.

the only unknown variable is now L (the length of the rope) :

3.4 = 2×pi × sqrt(L/9.8)

3.4/(2pi) = sqrt(L/9.8)

(3.4/(2pi))² = L/9.8

3.4²/(4pi²) = L/9.8

11.56/(4pi²) = L/9.8

2.89/pi² = L/9.8

9.8 × 2.89/pi² = L = 2.869618563... m ≈ 2.9 m

Write the recursive rule for the 5th term of the sequence: -7, -3, 1, 5, 9

Answers

Answer:

Step-by-step explanation:

Let  [tex]x_{n}[/tex] represent the nth term in the sequence. Then  [tex]x_{n-1}[/tex] represents the previous term

The first term, [tex]x_{1}[/tex] = -7

Each consecutive term is obtained by adding 4 to the previous term.

So the recursive relation is
[tex]\left \{ {{x_{1}= -7} \atop {x_{n}= x_{n-1} + 4}} \right.[/tex]

How many three-digit multiples of 5 have three different digits?

Answers

There are 180 numbers which are three-digit multiples of 5.

According to the statement

we have to find the three digit numbers which are multiples of 5.

we know that three digit numbers are start from 100 to 999 and in 1 line of counting means 100 to 110 there are 2 numbers are multiples of 5.

Here we use Multiplication method to find the number of digit which are a multiple of 5.

It means there are two numbers which are multiples of 5 in the one line of counting and there are 90 lines of  counting from 100 to 999.

So, it means the answer will become

2*90 = 180.

So, There are 180 numbers which are three-digit multiples of 5.

Learn more about NUMBERS here https://brainly.com/question/1094036

#SPJ1

write the equation of the line that is parallel to the line represented by 5 x + 2 y = 6 and passes through the point (1, 5.5).

Answers

Answer:

[tex]y=-2.5x+8[/tex]

Step-by-step explanation:

Parallel lines have the same slope but different y-intercepts. So, first, transform the given equation into the slope-intercept form, [tex]y=mx+b[/tex].

[tex]5x+2y=6[/tex]

[tex]2y=-5x+6[/tex]

[tex]y=-\frac{5}{2} x+3[/tex]

Then, determine the equation of a line that has a slope of [tex]-\frac{5}{2}[/tex] and passes through (1, 5.5). Substitute all the known values and solve for b.

[tex]y=mx+b[/tex]

[tex]5.5 = -\frac{5}{2} *1+b[/tex]

[tex]5.5=-2.5+b[/tex]

[tex]b=8[/tex]

Therefore, the answer is [tex]y=-2.5x+8[/tex].

Check the box labeled Show Segment Parallel to BC. Notice that intersects two sides of creating a smaller triangle, . How is related to ? How do you know?

Answers

Can i see an image of this problem

How do you classify the following polynomial?
-4x³ + 2x² - 5x+3x²

(I need de classification, no the result)

Answers

Cubic Because of the x^3 tells you by the power of the x

What is the complete factorization of the polynomial below?
x³ + 2x² + 4x+8
A. (x-2)(x+ 2i)(x+2i)
B. (x-2)(x+2i)(x-2i)
C. (x+2)(x + 2i)(x+2i)
D. (x+2)(x+2i)(x-2i)

Answers

The complete factorisation of the polynomial given; x³ + 2x² + 4x + 8 as in the task content can be factorised and determined as; Choice D; (x+2)(x+2i)(x-2i).

What is the complete factorisation of the given polynomial; x³ + 2x² + 4x+8?

It follows from the given task content that the polynomial whose factors are to be determined by means of factorisation is; x³ + 2x² + 4x+8.

It follows from observation that one of the zeros of the polynomial expression is at; x = -2.

Consequently, one of the factors of the polynomial in discuss is; (x+2).

x³ + 2x² + 4x+8 = (x+2) (x² - 4i²)

= (x+2)(x+2i)(x-2i)

Consequently, it follows that the complete factorisation of the polynomial is; (x+2)(x+2i)(x-2i).

Read more on polynomial factorisation;

https://brainly.com/question/11434122

#SPJ1

A group of college students are going to a lake house for the weekend and plan on renting small cars and large cars to make the trip. Each small car can hold 5 people and each large car can hold 7 people. A total of 10 cars were rented which can hold 66 people altogether. Write a system of equations that could be used to determine the number of small cars rented and the number of large cars rented. Define the variables that you use to write the system.

Answers

The system of equations that could be used to determine the number of small cars rented and the number of large cars rented is;

x + y = 10

x + y = 105x + 7y = 66

Simultaneous equation

Small car = 5 peopleLarge car = 7 peopleTotal cars = 10Total number of people = 66

Let

Number of small cars rented = x

Number of large cars rented = y

x + y = 10

x + y = 105x + 7y = 66

Learn more about simultaneous equation:

https://brainly.com/question/16863577

#SPJ1

¿cuál es el número que no pertenece a esta suceción 99,105111,117,123,129,135,141,147,153

Answers

No hay ninguno de los nimbers que no pertenezca a la secuencia.

¿Qué número no pertenece a la secuencia?

Sabemos que una secuencia tiene que ver con un arreglo de números en un orden definido. Todos los números en una secuencia deben tener la misma razón común o diferencia común.

Si miramos todos los números que están escritos en la secuencia, la diferencia común aquí es seis. Se sigue entonces que todos los números son parte de la sucesión y no hay ninguno de los números que no pertenezca a la sucesión.

Obtenga más información sobre la secuencia: https://brainly.com/question/21961097

#SPJ1

Need help with this math question

Answers

The correlation coefficient is -0.87; strong correlation

How to determine the correlation coefficient?

The given parameters are:

x = Time spent working out

y = lbs Overweight

Next, we enter the table of values in a graphing tool.

From the graphing tool, we have the following summary:

X Values

∑ = 27.1Mean = 2.71∑(X - Mx)2 = SSx = 22.569

Y Values

∑ = 89Mean = 8.9∑(Y - My)2 = SSy = 778.9

X and Y Combined

N = 10∑(X - Mx)(Y - My) = -114.19

R Calculation

r = ∑((X - My)(Y - Mx)) / √((SSx)(SSy))

r = -114.19 / √((22.569)(778.9))

r = -0.8613

Approximate

r = -0.87

This means that the correlation coefficient is -0.87

Also, the correlation coefficient is a strong correlation, because it is closer to -1 than it is to 0

Read more about correlation coefficient at:

https://brainly.com/question/27226153

#SPJ1

Jermy deposited money into a savings account that pays a simple annual interest rate of 1.6%. He earned 55.04 in interest after 4 years. How much did he deposit?.
860
1376
2976
3440

Answers

$860 is the principal amount. Therefore, choice A is right.

Simple interest rate is 1.6 percent.

$55.04 after simple interest.

t = 4 years

We must ascertain the principal sum.

Simple interest

The formula for simple interest is:
pxrxI
100
Substitute the given values.
D x 1.6x 4
55.04
100
5504 = 6.4p
5504
6.4
860 = 0
The principal value is $860. So, option A is
correct.

Answer: THE ANSWER IS A) $860

Step-by-step explanation: I GOT A 100 ON THE TEST

Choose the equation that matches
the graph.

a.y = 3x - 2
b. y = 3x-2
c. y = 3x+2
d. y = 3x + 2
e. y = (1/3)x-2

Answers

Answer:

it matches with equation a

Can someone please explain the steps to get too the second answer also please answer too thank you

Answers

Answer : y = 81
Explanation :
So y = x × 9. Then if x = 9, mutiply it by 9 and equals 81.
Hope it helps !

Christine has scored 73, 85, 70, and 63 on her previous four tests. What score does she need on her next test so that her average (mean) is 73?

Answers

Answer:

74

Step-by-step explanation:

(73 + 85 + 70 + 63 + x)/5 = 73.

You find the average by adding all of the test scores and dividing that number by the number of tests.  We had four test, so will will take that total, plus some mystery number (let's call it x), we divide that total by 5 because now there are 5 tests and not 4.  We already know what we want the average to be, we want the average to be 73.  We now have:

(291 + x)/5 = 73  Multiply both sides by 5

291 + x =365  Subtract 291 from both sides to get

x = 74

Answer: 74

Step-by-step explanation:

The mean is the average of all the values, which we find by finding the sum of each value and dividing by the total number of data points.

With 5 data points, we can use this equation to find, x, or the data point needed to obtain a mean of 73:

[tex]\frac{73+85+70+63+x}{5}=73\\ 291+x=365\\x=74[/tex]

Factor the expression.

3r2 – 75

Answers

Answer:

3(r + 5)(r - 5)

Step-by-step explanation:

Both 3 and 75 are divisible by 3, so we take out a 3.

3(r^2 - 25)

Next, we can factor r^2 - 25. It's a difference of squares.

3(r + 5)(r - 5)

Brainliest, please :)

To save $5,000 for a vacation in 3 years, how much money must be invested each
year if the investment earns 5% interest compounded annually? Round to the
nearest dollar.

$15,762
$1,586
$3,000
$1,667

Answers

[tex]~~~~~~ \textit{Compound Interest Earned Amount} \\\\ A=P\left(1+\frac{r}{n}\right)^{nt} \quad \begin{cases} A=\textit{accumulated amount}\dotfill & \$5000\\ P=\textit{original amount deposited}\\ r=rate\to 5\%\to \frac{5}{100}\dotfill &0.05\\ n= \begin{array}{llll} \textit{times it compounds per year}\\ \textit{annually, thus once} \end{array}\dotfill &1\\ t=years\dotfill &3 \end{cases}[/tex]

[tex]5000=P\left(1+\frac{0.05}{1}\right)^{1\cdot 3}\implies 5000=P(1.05)^3 \\\\\\ \cfrac{5000}{1.05^3}=P\implies 4319\approx P[/tex]

The amount to be invested annually to save $5,000 in 3 years, if the investment earns 5% annual interest, is $1,510.52.

How are annual payments or cash flows calculated?

The annual payments represent the periodic cash outflows invested in accumulating a future value using an interest rate compounded for a period.

This can be achieved using the future value and present value formulas or factors.

We can also use an online finance calculator, as follows:

Data and Calculations:

Future value = $5,000

Investment period = 3 years

Annual compound interest = 5%

N (# of periods) = 3 years

I/Y (Interest per year) = 5%

PV (Present Value) = $0

FV (Future Value) = $5,000 ($4,531.56 + $468.44)

Results:

PMT = $1,510.52

Sum of all periodic payments = $4,531.56 ($1,510.52 x 3_)

Total Interest = $468.44

Thus, the amount to be invested annually to save $5,000 in 3 years, if the investment earns 5% annual interest, is $1,510.52.

Learn more about periodic payments at https://brainly.com/question/24244579

I just need help on what the answer is and understanding this question, thank you.
The graph represents the potential area of a concrete rectangle, based on length and width.

Which inequality in vertex form represents the graphed region?

A)y less-than negative 2 (x + 16) squared minus 32
B)y less-than negative one-half (x minus 16) squared + 32
C)y less-than negative 2 (x minus 16) squared + 32
D)y less-than negative one-half (x + 16) squared minus 32

Answers

The inequality of the graph is y < -1/2(x - 16)^2 + 32

How to determine the inequality?

A quadratic function is represented as:

y = a(x - h)^2 + k

The vertex of the graph is

(h, k) = (16, 32)

So, we have:

y = a(x - 16)^2 + 32

The graph pass through the point

(x, y) = (12, 24)

So, we have:

24 = a(12 - 16)^2 + 32

Evaluate the like terms

-8 = a(-4)^2

This gives

16a = -8

Divide by 16

a = -1/2

Substitute a = -1/2 in y = a(x - 16)^2 + 32

y = -1/2(x - 16)^2 + 32

The graph is a less than graph.

So, we have

y < -1/2(x - 16)^2 + 32

Hence, the inequality of the graph is y < -1/2(x - 16)^2 + 32

Read more about inequality at:

https://brainly.com/question/17675534

#SPJ1

The total cost of two umbrellas and one cap is is $20 the total cost for one umbrella and two caps is $16
what is the price of each cap?
what is the price of what umbrella ?

Answers

If the total cost of two umbrellas and one cap is is $20 the total cost for one umbrella and two caps is $16 then the price of one umbrella is $8 and price of one cap is $4.

Given that the total cost of two umbrellas and one cap is is $20 the total cost for one umbrella and two caps is $16.

We have to find the price of one umbrella and one cap.

We have to form equations first to find the price of umbrella and cap.

Equation of two variables look like ax+by=c. It can be a linear equation, quadratic equation or a cubic equation.

let the price of umbrella be x and price of cap is y.

We know that total cost is equal to price of one unit multiplied by the units.

According to question.

2x+y=20----------1

x+2y=16-----------2

Multiply equation 2 by 2 and then subtract resulted equation from equation 1.

2x+y-2x-4y=20-32

-3y=-12

y=-12/-3

y=4

put the value of y in equation 2 to get the value of x.

x+2y=16

x+2*4=16

x+8=16

x=16-8

x=8

Hence the price of one umbrella is $8 and the price of one cap is $4.

Learn more about equations at https://brainly.com/question/2972832

#SPJ1

Find the area of a triangle with base of 6m and height of 15m

Answers

Answer:

45 meters

Step-by-step explanation:

[tex]Area=\frac{1}{2}bh\\ A=\frac{1}{2}(6)(15)\\ A=3(15)\\A=45[/tex]

⊱________________________________________________________⊰

Answer:

A=45 m²

Step-by-step explanation:

The formula for the area of a triangle is, [tex]\bigstar\underline{\boxed{\sf{A=\frac{bh}{2}}}}[/tex].

where :

[tex]\triangleright\sf{A=Area,b=base,h=height}\triangleleft[/tex]

Substitute the values,

[tex]\large\begin{gathered}\sf{A=\frac{6*15}{2} \\ \sf{A=\frac{90}{2}\\\bigstar\underline{\boxed{\sf{A=45 \ m^2}}}} \end{gathered}[/tex]

Done !! Hope this made sense to you :)

[tex]\small\boldsymbol{Calligraphy}[/tex]

⊱______________________________________________________⊰

Find the decimal. _. _ _ _ Clues: (a) The digits 0, 1 and 2 are not used in the decimal. (b) The digit in the ones place is the greatest 1-digit number. The hundredths digit is smaller than 5. The difference between the tenths digit and the ones digit is the hundredths digit. (c) (d) (e) (f) The thousandths digit is greater than 5 and is an odd number. All the digits are different.​

Answers

Answer:

Step-by-step explanation:

143

or

121

We are accepting

does any of this help

NO LINKS!!! Please help me with this problem​

Answers

Answer:

length: 8end points: (-8, 1), (0, 1)

Step-by-step explanation:

In the quadratic form ...

  (x -h)² = 4p(y -k)

The value 4p is the length of the latus rectum. It also tells the distance between the vertex (h, k) and the focus (h, k+p).

Application

The value of 4p is given as 8. This means p = 8/4 = 2.

The length of the latus rectum is 8.

The end points of the latus rectum have the same y-value as the focus:

  y = k +p = -1 +2 = 1

The end points of the latus rectum have x-values that are h±2p.

  x = h ± 2p = -4 ± 2·2 = {-8, 0}

The end points of the latus rectum are (-8, 1) and (0, 1).

Heights (cm) and weights (kg) are measured for 100 randomly selected adult males, and range from heights of 130 to 190 cm and weights of 40 to 150 kg. Let the predictor variable x be the first variable given. The 100 paired measurements yield x= 167.65 cm, y= 81.36 kg, r= 0.309, P-value= 0,002, and y= 106+1.15x. Find the best predicted value of y (weight) given an adult male who is 156 cm tall. Use a 0.10 significance level.

Answers

The best predicted weight of a person that is 156 cm is 285.4kg

We have the regression equation as

y= 106+1.15

When the adult male is 156 cm tall the weight of this person would be calculated as:

y= 106+1.15*156

y = 106 + 179.4

y = 285.4

Hence we can arrive at the conclusion that the best predicted weight of a person that is 156 cm is 285.4kg

Read more on regression equation here:

https://brainly.com/question/26843436

#SPJ1

The height "h" of a ball thrown straight up with a velocity of 88 ft/s is given by h = -16t^2 + 88t where "t" is the time it is in the air. For how many seconds the ball is in the air before it hits the ground?

Answers

By definition of the zeros of ta quadratic function,  for 5.5 seconds the ball is in the air before it hits the ground.

Zeros of a function

The points where a polynomial function crosses the axis of the independent term (x) represent the so-called zeros of the function.

That is, the zeros represent the roots of the polynomial equation that is obtained by making f(x)=0.

In summary, the roots or zeros of the quadratic function are those values ​​of x for which the expression is equal to 0. Graphically, the roots correspond to the abscissa of the points where the parabola intersects the x-axis.

Time the ball is in the air before it hits the ground

In this case, the height "h" of a ball thrown straight up with a velocity of 88 ft/s is given by h = -16t² + 88t, where "t" is the time it is in the air.

When the ball hits he ground, he height h has a value of zero. This is h=0. Replacing in the previous expression for the height you get:

0= -16t² + 88t

It can be solved by extracting the term "t" as a common factor:

0= t×(-16t + 88)

The Zero Product Principle says that if the product of two numbers is 0, then at least one of the factors is 0. Then:

t= 0

or

0= -16t + 88

Solving: -88= -16t

(-88)÷ (-16)=t

5.5= t

Finally, this means that for 5.5 seconds the ball is in the air before it hits the ground.

Learn more about the zeros of a quadratic function:

brainly.com/question/12649112

brainly.com/question/21320065

brainly.com/question/12037466

#SPJ1

4. (a)(i)Show that log4x=2log16x. (ii)Show that log x=3logb³ x. (iii) Show that log₂x=(1+log₂3)logix.​

Answers

that log4x=2log16x. (ii)Show that log x=3logb³ x. (iii) Show that log₂x=(1+log₂3)logix

correct the error 4⁴- 4³=4¹​

Answers

The error is that

[tex] {x}^{y} - {x}^{z} \neq {x}^{y - z} [/tex]

Instead, it should be 4⁴ - 4³ = 256 - 64 = 192.

Other Questions
Complemento directo e indirectoContesta las preguntas sustituyendo las palabras subrayadas con pronombres de complemento directo e indirecto.(MODELO) Me prestas el libro?S, te lo presto.1. Tiene Gabriel que ver esa obra de teatro?No, no _________________2. Siempre celebran tus padres su aniversario?S, siempre ______________3. Nos va a recomendar Marta estas pelculas?No, no ____________________4. Me das tu opinin sobre el espectculo?S, _________________________5. Vas a comprarle esos CD a Emilia?S, __________________________6. Quieren ganar ustedes este partido?S, ___________________________ 1. Name a career that requires data-analysis skills. Describe how data analysis isused in this career. Be specific in your examples. Darker colored rocks that are rich in iron and magnesium are are referred to as being ____________________ in composition. Which people or institutions have the right to use electronic health records? a. friends b. pharmacies c. support groups d. medical blogs Phenytoin blocks the opening of voltage-gated na channels in neurons. therefore, this action has its primary effects on which part of the neuron? A bag contains 1 white (W), 3 blue (B1, B2, B3), and 2 red (R1, R2) marbles.Find the probability of tossing a head and drawing a red marble. Explain your reasoning. According to the "Law of Increasing Opportunity Costs," what would be the opportunity cost of a student who is staying up all night to study for an exam that he has to take in the early morning? A. Food or Drink. B. Money or income. C. Sleep or rest. Please answer hurry! A _____ is an interval estimate of an individual y value, given values of the independent variables. Group of answer choices prediction interval confidence interval interval estimation regression In a certain card game, the probability that a player is dealt a particular hand is. Explain what this probability means. If you play this card game 100 times, will you be dealt this hand exactly times? why or why not?. how many positive odd factors of 48 are greater than 2 and less than 10 John Daum and Chris Yin are star swimmers at a local college. They are preparing to compete at the NCAA Division II national championship meet, where they both have a good shot at earning a medal in the mens 100-meter freestyle event. The coach feels that Chris is not as consistent as John, even though they clock about the same average time. In order to determine if the coachs concern is valid, you clock their time in the last 9 runs and compute a standard deviation of 0.93 seconds for John and 1.08 seconds for Chris. It is fair to assume that clock time is normally distributed for both John and Chris. Let the clock time by John and Chris represent population 1 and population 2, respectively.Calculate the value of the test statistic. find x and y can someone pls solve normally how often should B-12 supplement be used for energy? Find the missing length indicated. i need help with steps Calculate the perimeter and area of this shape.Perimeter =Area =7.5 cm6 cm2 cm3 cm2 cm Although Thelonious Monk was an innovator known as The High Priest of Bop, Dizzy Gillespie credited Bud Powell, not Monk, with inventing many of the harmonic principles of bebop. Another name for a negotiated labor-management agreement is a? A triangle is given by A ( 3,2 ) B (-1,5) and C (0,-1) . What is the equation of the line through B parallel to AC ( give ans and how it works pls ) What was the purpose of the proposed anti-lynching legislation that President Wilson refused to sign?to help local police to enforce existing lawsto reduce lynching from a felony to a misdemeanorto encourage African Americans to sue their attackersto criminalize mob executions